$\begingroup$

The gradient of the curve $=\frac{a}{10}+bx^2$ at the point (iii,6) is 7. Calculate the values of a and b.

I did it,
$half-dozen=\frac{a}{3}+b(three^ii) \tag{1} $ We also have: (derivative) $y'=-\frac{a}{ten^2}+2bx \tag{ii}$
$7=-\frac{a}{3^ii}+2b(3) \tag{3} $
but it doesn't seen right.

the answer is a=-9, b=1

Can you lot help me out? cheers.

asked April 13, 2012 at 1:28

Cin Sb Sangpi's user avatar

$\endgroup$

5

  • $\begingroup$ In equation $(1)$ a $b$ is missing. $\endgroup$

    user21436

    Apr 13, 2012 at one:34

  • $\begingroup$ Your values of $a$ and $b$ are right. $\endgroup$

    user21436

    April 13, 2012 at 1:36

  • $\begingroup$ Why practise you think you lot could be wrong? Anything particular you're doubtful almost? $\endgroup$

    user21436

    Apr 13, 2012 at one:38

  • $\begingroup$ yep, i don't know how to exercise farther. thx :) $\endgroup$

    April 13, 2012 at i:42

  • $\begingroup$ Information technology was not clear if yous wanted us to check the answer or how to get that answer? Oh, well, there are answers now. $\endgroup$

    user21436

    Apr 13, 2012 at 1:44

ii Answers two

$\begingroup$

From $half dozen=\frac a3+9b$ it follows that $a=18-27b$. Substituting into (3) we get $seven=-\frac{eighteen-27b}{9}+6b=3b-two+6b$, hence $b=i$ and $a=eighteen-27=-9$.

answered Apr 13, 2012 at one:38

azarel's user avatar

$\endgroup$

6

  • $\begingroup$ At the time of this annotate, your reputation is $3935$, and mine is $3539$... interesting! $\endgroup$

    April 13, 2012 at 1:43

  • $\begingroup$ @TheChaz A fleck strange indeed. $\endgroup$

    Apr 13, 2012 at 1:47

  • $\begingroup$ how did $a=18-27b$, i got only $18=a+27b$, thx. $\endgroup$

    Apr 13, 2012 at 1:58

  • $\begingroup$ Sb, if you lot don't know how to go from $$eighteen = a + 27b$$ to $$a = 18 -27b$$, and so nosotros might take a trouble! $\endgroup$

    April 13, 2012 at 2:01

  • $\begingroup$ aww..i see, so sorry, $a+27b=18$ and so $a=eighteen-27b$ $\endgroup$

    Apr 13, 2012 at 2:05

$\begingroup$

If you multiply equation (3) by 3, so add information technology to equation (1), yous'll get $$27 = 27b$$

From this, the value of $a$ follows.

answered Apr xiii, 2012 at 1:40

The Chaz 2.0's user avatar

$\endgroup$